Displaying 171-180 of 529 result(s).

SAT Question of the Day

2012-09-10 09:00:49

The music scholar was unique in his profession, for, unlike other
(A)
scholars, he preferred Bach to Beethoven and would often make the
(B) (C)
argument that the former was the best of the two composers. No error
(D) (E)

The correct answer is D

Explanation: The error in this sentence occurs at choice D where a superlative adjective is incorrectly used to describe one of two entities. When only two things are compared, a comparative adjective needs to be used. To correct this comparison/modifier error, one could change "best" to "better."

SAT Question of the Day

2012-09-12 09:00:51

Triangle FGH has one leg measuring 14 units and one leg measuring 22 units. Which of the following could be the perimeter of FGH?

I.  44 units
II. 64 units
III.  74 units

(A) I only
(B) II only
(C) I and II
(D) II and III
(E) I, II, and III

The correct answer is B

Explanation: The third side of a triangle has to be less than the sum of and greater than the difference between the other two sides. Thus, the third side of ∆FGH must be longer than 8 units and less than 36 units (since 22 - 14 = 8, and 14 + 22 = 36). The perimeter is the distance around the outside of an object, so the perimeter here is the sum of the three sides. Two of those sides add up to 36, and the third side must be between 8 and 36, meaning the perimeter must be between 36 + 8 and 36 + 36. Thus, if P is perimeter, 44 < P < 72. Of the answer choices, only 64 is in that range, so only Roman Numeral II provides a number that COULD be the perimeter of ∆FGH.

SAT Question of the Day

2012-09-14 09:00:29

Rick proved ______ when it came to planning for the future, and this lack of ______ proved costly when he ran out of money after just two years of retirement.

(A) myopic...foresight
(B) improvident...clairvoyance
(C) sagacious...frugality
(D)miserly...wisdom
(E) prudent...economy

The correct answer is A

Explanation: To find the missing words, use keywords in the prompt to predict their meaning. In this prompt, the main clue is the word "and" which links the two clauses. "And" is used to link clauses that state similar or related information, which means there must be a relationship between what Richard was and the thing he lacked. This relationship implies that the words must be opposites, since something Richard has would be the opposite of what he lacks. The prompt provides one additional clue in stating that Richard ran out of money. Thus, Richard must have been incautious and lacked the ability to save money. Choice A is correct, then: "Myopic" means "lacking foresight or the ability to act prudently," while "foresight" is "care or provision for the future." Thus, because Rick was myopic, he lacked foresight and lost his money.

None of the other answer choices work, either:

improvident: not having or showing foresight
clairvoyance: the ability to predict or see the future (the first word would make sense in context, but "clairvoyance" is too literal to work)

sagacious: shrewd or wise
frugality: the quality of being economical with money (the second word would make sense in context, but if Rick were "sagacious," he probably would not lack "frugality")

miserly: hoarding of money
wisdom: the quality of having good judgment (the second word could work in context, but if Rick were "miserly," he would probably not run out of his money after just two years of retirement)

prudent: acting with care for the future
economy: careful management of money or resources (the second word could work in context, but if Rick were "prudent," he would have "economy," not lack it)

SAT Question of the Day

2012-09-17 09:00:36

Five consecutive integers are selected and have a median of x. Which of the following CANNOT affect their median?

(A) Multiplying all the numbers by their average
(B) Subtracting the third highest number.
(C) Adding five to all the numbers
(D) Dividing all of the numbers by x
(E) Adding another number to the set

The correct answer is B

Explanation:

In a set of consecutive integers that contains an odd number of integers, the middle number will be the same as the median. If the middle number is removed, though the median will not be affected, as the median will then be the average of the middle two numbers (since there would be only four numbers). The average of two numbers that are two integers apart is the integer that would be between them numerically (meaning the average of 2 and 4 is 3, the average of -1 and 1 is 0, the average of 100 and 102 is 101, etc.).

SAT Question of the Day

2012-09-19 09:00:39

Though he was ultimately overruled, Winston Churchill first suggested that the American and British joint command focus their war efforts on an invasion of Italy, rather than on the invasion of France that American leaders had been clamoring for since 1942.

(A) Winston Churchill first suggested that the American and British joint command focus their war efforts on an invasion of Italy
(B) the war effort of the joint American and British command should have focused on invading Italy, Winston Churchill suggested
(C) Winston Churchill first suggested that the American and British joint command focus its war efforts on an invasion of Italy
(D) Winston Churchill first suggested that the Americans and British joint command focus its war efforts on invading Italy
(E) American and British joint command, Winston Churchill suggested, should focus its war effort on invading Italy

The correct answer is C

Explanation: The mistake in this sentence occurs at the word "their," a plural possessive pronoun that does not agree with its singular antecedent, the collective noun clause "American and British joint command." Choice (C) corrects this mistake by changing "their" to the proper, singular "its."

SAT Question of the Day

2012-09-26 09:00:51

A cube has a volume of x3. If the height of the cube is doubled, the surface area of the resulting rectangular solid will be which of the following?

(A) 2x3
(B) 4x2 + 2x3
(C) 6x2
(D) 10x2
(E) 4x3 + 2x2



The correct answer is D

Start by finding the current lengths of the cube. If it has a volume of x3, then each side must measure x (since a cube's volume is just the length of one side of the cube cubed). If the height is doubled, then it will measure 2x in the new solid. Surface area of a rectangular prism is 2lw + 2lh + 2wh (the sum of the areas of each face). l and w will both be x here, but h will be 2x. Thus, the surface area of the figure will be 2(x)(x) + 2(x)(2x) + 2(x)(2x). This simplifies to 2x² + 4x² + 4x², which sums to 10x².

SAT Question of the Day

2012-09-28 09:00:47

Calvin Coolidge was known for being quite terse; when a
(A)
woman approached him at a banquet and told him she had bet her
(B)
friend she could get him to say more than two words, it was not
(C)
unusual that he replied simply, "You lose." No error
(D) (E)




The correct answer is E

Explanation: There is no error in this sentence.

There is no error at choice (A) because "when" is an appropriate preposition to use when describing time, and the semicolon before the "when" prevents the two independent clauses from creating a run-on.

There is no error at choice (B) because the past tense verb "approached" is in the proper tense and agrees with its subject, "a woman" in number. Moreover, "him" is an appropriate pronoun to use, as it is in the proper objective case and has a clear antecedent in Coolidge.

There is no error at choice (C) because the comparative adjectival phrase "more than two" correctly modifies the noun "words."

There is no error at choice (D) because the adverb "simply" correctly modifies the verb "replied."

SAT Question of the Day

2014-03-19 09:00:17

In countries so poor that they can barely pay their police,  officers are notoriously ______, with many of them willing to look the other way on almost any criminal activity for the right price.

(A) venal
(B) vitriolic
(C) venomous
(D) vituperative
(E) venerable

The correct answer is A
Explanation: To find the missing word, try to predict its meaning by using keywords and clues in the sentence. Here, the best clues are that the police are "barely" able to be paid and are, as such, "willing to look the other way... for the right price." This suggests that the police are willing to take money in exchange for abuses of their power, implying that they are easily bribed. The missing word describes what the police are, so it follows that it too must mean "easily bribed," making choice A correct: "venal" means "showing a willingness to accept bribes or motivated by bribery."

None of the other choices work, either:

vitriolic: filled with malice
venomous: full of malice
vituperative: bitter and abusive
venerable: laudable or praiseworthy

SAT Question of the Day

2014-03-21 09:00:50

Set A consists of all the integers that are greater than 9 and less than 100.
Set B consists of all two-digit even integers.

What is the difference between the sum of all the numbers in Set A and the sum of all the numbers in set B?

The correct answer is 2475
Explanation: There is no need to add together both sets of numbers. As Set B will be subtracted from Set A in the end, you may as well just subtract it from Set A in the beginning. That is, eliminate from Set A all the numbers that are included in Set B, which is all the even numbers in set B. Thus, what is left are just the odd numbers between 11 and 99, inclusive.

Now, add the endpoints together, pairing them off one by one:
11 + 99 is 110
13 + 97 is 110
15 + 95 is 110
17 + 93 is 110... see the pattern?

Thus, you merely need to figure out the number of pairs that will be formed and multiply it by 110.
All the numbers in the tens will pair will all the numbers in the 90s, to create 5 pairs.
All the numbers in the 20s will pair will all the numbers in the 80s, to create 5 pairs.
All the numbers in the 30s will pair will all the numbers in the 70s, to create 5 pairs.
All the numbers in the 40s will pair will all the numbers in the 60s, to create 5 pairs.
Then, for the 50s, there will be
51 + 59
53 + 57
and 55 left all alone.

Thus, there are 22 pairs that sum to 110, and one individual 55. 22 x 110 = 2420. 2420 + 55 is 2475.

SAT Question of the Day

2014-03-26 09:00:00

The vertex of an isosceles triangle measures 4a - 40 degrees. If one of the base angle measures 3a + 15 degrees, what is the value of a?

(A) 15
(B) 19
(C) 25
(D) 36
(E) 72

The correct answer is B
Explanation: Since the triangle is isosceles, the two base angles must have the same measure, meaning both are 3a + 15. All three angles must sum to 180, so 4a - 40 + 3a + 15 + 3a + 15 = 180. Thus, 10a -10 = 180. Move the 10 over to get 10a = 190. Divide both sides by 10 to get a = 19. Thus, B is correct.